5
$\begingroup$

From Terry Tao's post here there is the statement:

"Conversely, if one can somehow establish a bound of the form

$$\displaystyle \sum_{n \leq x} \Lambda(n) = x + O( x^{1/2+\epsilon} ) \tag{1}$$

for any fixed ${\epsilon}$, then the explicit formula can be used to..."

I don't know about the word "fixed", but the irregular behaviour of the blue curve below gives plenty of room for an ${\epsilon}$, if it is true that the asymptotic is $(8x)^{1/2}$, and if it is also true that it bounds the partial sums of the Möbius transform of the Harmonic numbers minus $x$. But we don't know and can't conclude any such bounds from this question. I am only asking about the asymptotics of a certain sum that is connected to / a truncated absolute value version of the numerators of the expansion of the primes.

Let:

$$\varphi^{-1}(n) = \sum_{d \mid n} \mu(d)d \tag{2}$$

Then for $n>1$:

$$\Lambda(n) = \sum\limits_{k=1}^{\infty}\frac{\varphi^{-1}(\gcd(n,k))}{k} \tag{3}$$

Form the table: $$A(n,k)=\sum_{\substack{i=k\\\ n \geq k}}^n \varphi^{-1}(\gcd (i,k)) \tag{4}$$

From numerical evidence it appears that:

$$\sum _{k=1}^{x} \text{sgn}\left(\left(\text{sgn}\left(x+\sum _{j=2}^k -|A(x,j)|\right)+1\right)\right)+1 \sim (8x)^{1/2} \tag{5}$$

Is it true or is the asymptotic something else?

Question:

The complicated sign formula in $(5)$ comes from what we are really doing which is to ask: What is the asymptotic of the least $k$ for which the function $F(x)$:

$$F(x)=x+\sum _{j=2}^k -|A(x,j)| \tag{6}$$

is negative? For $k=1..x$.

Plot of the numerical evidence where the irregular blue curve is that least $k$ for which the function $F(x)$ is negative and thereby also the LHS of (5) while the smooth red curve is the conjectured asymptotic $(8x)^{1/2}$:

what asymptotics sqrt 8x

Efficient Mathematica program to generate the plot. Setting nn=10000 gives the plot above:

(*start*)
(*Mathematica*)
Clear[a, f, p];
nn = 1000;
p = 0;
f[n_] := n*Log[n]^p;
(*Clear[f];*)
(*f[n_] := n*Log[n]^4/(Pi*8)^2/8;*)
a[n_] := DivisorSum[n, MoebiusMu[#] # &];
Monitor[TableForm[
   A = Accumulate[
     Table[Table[If[n >= k, a[GCD[n, k]], 0], {k, 1, nn}], {n, 1, 
       nn}]]];, n]
TableForm[B = -Abs[A]];
Clear[A];
B[[All, 1]] = N[Table[f[n], {n, 1, nn}]];
TableForm[B];
TableForm[B1 = Sign[Transpose[Accumulate[Transpose[B]]]]];
Clear[B];
Quiet[Show[
  ListLinePlot[
   v = ReplaceAll[
     Flatten[Table[First[Position[B1[[n]], -1]], {n, 1, nn}]], 
     First[{}] -> 1], PlotStyle -> Blue], 
  Plot[Sqrt[8*f[n]], {n, 1, nn}, PlotStyle -> {Red, Thick}], 
  ImageSize -> Large]]
ListLinePlot[v/Table[Sqrt[8*f[n]], {n, 1, nn}]]
(*end*)

Variant of the Mathematica program above: https://pastebin.com/GJ81MQez

Inefficient Mathematica program to generate the LHS in (5):

Clear[varphi];
nn = 20;
constant = 2*Sqrt[2];
varphi[n_] := Total[Divisors[n]*MoebiusMu[Divisors[n]]];
Monitor[TableForm[
   A = Table[
     Table[Sum[If[n >= k, varphi[GCD[i, k]], 0], {i, k, n}], {k, 1, 
       nn}], {n, 1, nn}]];, n]
Table[1 + 
  Sum[Sign[(1 + Sign[x + Sum[-Abs[A[[x, j]]], {j, 2, k}]])], {k, 1, 
    x}], {x, 1, nn}]

which starts: {2, 3, 4, 5, 6, 5, 7, 7, 10, 7, 11, 10, 11, 10, 11, 11, 14, 13, 14, 13}

For my own memory to remember where to start editing tomorrow I write this Mathematica program:

Clear[varphi];
nn = 40;
varphi[n_] := Total[Divisors[n]*MoebiusMu[Divisors[n]]];
Table[1 + 
  Sum[Sign[(1 + 
      Sign[x + 
        Sum[-Abs[
           Sum[If[x >= j, varphi[GCD[i, j]], 0], {i, j, x}]], {j, 2, 
          k}]])], {k, 1, x}], {x, 1, nn}]

There are previous efforts related to this question. Here is one of them.

A construction: $$\sqrt{x} \log ^2(x)=\sqrt{x} \left(x-\left(\sqrt{x}-\log (x)\right) \left(\sqrt{x}+\log (x)\right)\right)$$

$\endgroup$
6
  • $\begingroup$ Proof of von Mangoldt function formula: math.stackexchange.com/a/51708/8530, mathoverflow.net/a/162214/25104 $\endgroup$ May 1, 2020 at 11:05
  • $\begingroup$ Sqrt(8) in the OEIS: oeis.org/A010466 $\endgroup$ May 1, 2020 at 11:08
  • 2
    $\begingroup$ Could you say clearly if the "blue curve" represents $\sum_{n \leq x} \Lambda(n) -x$ or something else? $\endgroup$
    – YCor
    May 28, 2020 at 20:11
  • $\begingroup$ The irregular blue curve is not $\sum_{n \leq x} \Lambda(n) -x$. I will try to edit and clarify. It uses negated absolute values of the terms in the same expansion as $\sum_{n \leq x} \Lambda(n)$ though. $\endgroup$ May 28, 2020 at 20:33
  • 3
    $\begingroup$ You don't have to give a complicated formula with signs, you can just say "Let $F(n)$ be the least $k$ such that .... is negative" $\endgroup$
    – Will Sawin
    May 28, 2020 at 21:02

3 Answers 3

12
$\begingroup$

Let us denote the left hand side of $(1)$ by $\psi(x)$. It is known that $|\psi(x)-x|$ is not bounded by a constant times $x^{1/2}$. In fact Littlewood (1914) proved that $$\psi(x)-x=\Omega_{\pm}(x^{1/2}\log\log\log x).$$ This is Theorem 15.11 in Montgomery-Vaughan: Multiplicative number theory I.

$\endgroup$
2
$\begingroup$

If one rewrites $(4)$ as: $$A(n,k)=\sum_{i=1}^n \varphi^{-1}(\gcd (i,k))$$ Then one finds empirically that the mean of the $k$-th column in $A(n,k)$ is:

$$\lim_{n\to\infty} \frac{1}{n}\sum_{k=1}^n A(n,k) = -\frac{\varphi^{-1}(k)}{2} \tag{*}$$

and that the period length of column $k$ is $k$.

In https://oeis.org/A173557 on Jun 18 2020, Vaclav Kotesovec says that the Dirichlet generating function for: $$a(k)=|\varphi^{-1}(k)|$$ is: $$\frac{\zeta(s)\zeta(s-1)}{\zeta(2s-2)} \prod_{p \text{ prime}} \left(1 - \frac{2}{(p + p^s)}\right)$$

and per email he explained to me that by choosing the residue at $s=2$ he finds that:

$$\sum_{k=1}^{n} a(k) \sim c \frac{n^2}{2}$$ or equivalently: $$\sum_{k=1}^{n} |\varphi^{-1}(k)| \sim c \frac{n^2}{2} \tag{**}$$

where: $$c = A307868 = \prod_{p \text{ prime}} \left(1-\frac{2}{(p+p^2)}\right) = 0.471680613612997868...$$

In the question above we ask what is the least $k$ for which the function $F(n)$: $$F(n)=n-\sum _{j=2}^k|A(n,j)|$$ becomes negative. Since the columns are periodic and the average of the periods in the $k$-th column is as said above: $-\frac{\varphi^{-1}(k)}{2}$, this is then approximately equal to asking what is the least $k$ for which the function $G(n)$: $$G(n)=n-\sum _{j=2}^k\frac{|\varphi^{-1}(j)|}{2} \tag{***}$$ becomes negative.

Combining $(*)$ and $(**)$: $$\sum _{j=1}^k\frac{|\varphi^{-1}(j)|}{2} \sim c\frac{k^2}{4}$$ Setting $j=2$ in the lower summation index: $$\sum _{j=2}^k\frac{|\varphi^{-1}(j)|}{2} \sim c\frac{k^2}{4}-1$$ Inserting into $(***)$:

$$G(n)=n-\left(c\frac{k^2}{4}-1\right)$$

The least $k$ for which $G(n)$ becomes negative or changes sign is when $G(n)=0$. Solving:

$$c\frac{k^2}{4} - 1 = n$$ for $k$ gives the answer:

$$k(n) = \frac{2 \sqrt{n+1}}{\sqrt{c}} = \sqrt{8.4803146 (n+1)}$$

where as the conjectured asymptotic for the least $k$ when $F(n)$ becomes negative was:

$$k(n) \sim \frac{2 \sqrt{n}}{\sqrt{\frac{1}{2}}}=\sqrt{8n}$$ for comparison.

$\endgroup$
1
  • $\begingroup$ Clear[nn, t, n, k]; b = {1, -1}; nn = 1000; a[n_] := Total[MoebiusMu[Divisors[n]]*Divisors[n]]; Monitor[ g1 = ListLinePlot[ Accumulate[ Table[Sum[ If[Mod[k, 2] == 0, 1, b[[1 + Mod[n, 2]]]]*a[GCD[n, k]]/N[k], {k, 1, n}], {n, 1, nn}]]], n]; g2 = Plot[(Log[n] - Log[2])^2*Log[2] - Log[2]/2, {n, 1, nn}, PlotStyle -> Red]; Show[g1, g2] $\endgroup$ Aug 3, 2023 at 22:13
0
$\begingroup$

Setting:

$$F(x)=x\log(x)+\sum _{j=2}^k -|A(x,j)| \tag{1}$$

appears to give the asymptotic $\sqrt{8x\log(x)}$ for the least $k$ such that $F(x)$ is negative.

In general it appears that the least $k$ such that:

$$F(x)=f(x)+\sum _{j=2}^k -|A(x,j)| \tag{2}$$

is negative, has the asymptotic: $\sqrt{8f(x)}$.

See the Mathematica program in the question, by setting p=1.

$\endgroup$
2
  • $\begingroup$ Clear[nn, t, n, k]; nn = 1000; a[n_] := Total[MoebiusMu[Divisors[n]]*Divisors[n]]; Monitor[g1 = ListLinePlot[ Accumulate[ Table[Sum[ If[Mod[k, 2] == 0, 1, (-1)^n]*a[GCD[n, k]]/N[k], {k, 1, n}], {n, 1, nn}]]], n]; g2 = Plot[(Log[n] - Log[2])^2/(1.45), {n, 1, nn}, PlotStyle -> Red]; Show[g1, g2, PlotRange -> {0, 30}] $\endgroup$ Aug 3, 2023 at 19:50
  • $\begingroup$ Monitor[g1 = ListLinePlot[ Accumulate[ Table[Sum[ If[Mod[k, 2] == 0, 1, (-1)^n]*a[GCD[n - k, k + n]]/N[k], {k, 1, n}], {n, 1, nn}]]], n] $\endgroup$ Aug 3, 2023 at 19:51

Your Answer

By clicking “Post Your Answer”, you agree to our terms of service and acknowledge you have read our privacy policy.

Not the answer you're looking for? Browse other questions tagged or ask your own question.